Induktiver Beweis einer Ungleichung

Neue Frage »

Christian_P Auf diesen Beitrag antworten »
Induktiver Beweis einer Ungleichung






Jetzt habe ich die Ungleichung mit der Zahl multipliziert, denn das ist jeweils der nächste Faktor für




Kann man damit argumentieren, dass der Bruch ist bzw. der Zähler kleiner als der Nenner ist und sich deshalb die linke Seite der Ungleichung verkleinert?

Die rechte Seite hab ich umgeformt zu:

Hier würde sich die Zahl vegrößern, erst recht, wenn: also im Indutkionsschritt.




Habt ihr einen Tipp?


Grüße
SusiQuad Auf diesen Beitrag antworten »
RE: Induktiver Beweis einer Ungleichung
Tipp:


Letzteres weil

HTH Wink
Christian_P Auf diesen Beitrag antworten »

Hi SusiQuad, danke für die Antwort.

Nur leider verstehe ich nicht ganz, was du genau meinst, und wie ich das nun auf meine Überlegungen anwenden kann.


liebe Grüße
HAL 9000 Auf diesen Beitrag antworten »

Zitat:
Original von Christian_P
und wie ich das nun auf meine Überlegungen anwenden kann.

Seltsame Frage. Denn abgesehen von dem kleinen Schreibfehler

Zitat:
Original von SusiQuad
Letzteres weil

hier müsste nämlich stehen, hat SusiQuad bereits den kompletten Induktionsschritt genannt.
Mystic Auf diesen Beitrag antworten »

Ja, wobei das Ausrechnen von (2n+1)(2n+3) sowieso kontraproduktiv ist (SusiQuad hat sich ja auch prompt dabei verrechnet!), sondern man sollte das in der Form ((2n+2)-1)((2n+2)+1) anschreiben und dannn die 3. Binomische Formel anwenden oder sich noch besser gleich auf die Tatsache berufen, dass das arithmetische Mittel zweier Zahlen niemals kleiner als das geometrische ist... Augenzwinkern
Christian_P Auf diesen Beitrag antworten »

Hi

Ich habe gerade knapp einen Monat Mathestudium (1.Semester) hinter mir. Dies nur am Rande Augenzwinkern


Von daher kann ich einfach noch nicht so viele Tricks anwenden. Mit Abschätzungen tue ich mich noch echt schwer.


Ich habe versucht zu begründen, warum die Ungleichung im Schritt von n nach n+1 immer noch gilt, indem ich mir die linke und rechte Seite angeschaut habe, in der Hinsicht, wie sich die Zahlen jeweils ändern. Nun hatte ich festgestellt, das die linke Seite kleiner wird und die rechte Seite vermutlich größer. Ganz entsprechend der Monotonie der Ungleichung.

Durch die Umformung:



hab ich dann für mich erkannt, dass die rechte Seite in jedem Fall größer wird, erst recht dann, wenn:

gilt.


Und jetzt kommt die Erklärung des Induktionsschrittes von SusiQuad ins Spiel, die ich gerade versuche genau zu verstehen. Das ist mathematisch natürlich viel cleverer, aber eben nicht ganz leicht.


Grüße
 
 
HAL 9000 Auf diesen Beitrag antworten »

Zitat:
Original von Christian_P
Durch die Umformung:



hab ich dann für mich erkannt, dass die rechte Seite in jedem Fall größer wird, erst recht dann, wenn:

gilt.

Du drückst dich undeutlich aus: Was wird größer mit dem "erst recht" ? Du hast dann nur noch einen Term genannt, aber nicht, wie der in irgendeine Abschätzung eingehen soll. unglücklich
Christian_P Auf diesen Beitrag antworten »



letzteres weil:




Ok jetzt seh ichs. Finger1

Rationalmachen des Nenners und dann die rechte Ungleichung umgeformt und quadriert. Hat'n bisschen gedauert Augenzwinkern Ja, dass ist ein bisschen einfacher umgeformt und begründet als meine Versuche. wunderbar!



Danke Euch allen.
Neue Frage »
Antworten »



Verwandte Themen

Die Beliebtesten »
Die Größten »
Die Neuesten »